Sie sind auf Seite 1von 1

signup

TeXLaTeXStackExchangeisaquestionandanswersiteforusersofTeX,LaTeX,ConTeXt,andrelated
typesettingsystems.It's100%free,noregistrationrequired.

login

How to insert overlays in latex beamer - TeX - LaTeX Stack Exchange

2/8/2014

tour

help

Takethe2minutetour

How to insert overlays in latex beamer


Ihaveasmanyas1520overlayswithina frame.Iamtryingtohighlightcertainstatesinastatediagram.TheproblemissometimesI
wouldwanttoaddanoverlay,saybetween5and6.Iwouldneedtoincreasethecounterofalltheoverlaysfollowing6whichcanbequite
annoying.Iconsiderednumberingoverlaysinmultiplesof5justsothatIcaninsertanoverlaylater.However,eachnumberwouldresultina
newslidewhichishighlyundesirable.Anothersolutioniscreatingaseparateframe.Isthereabetterwaytohandlethis?AmImissing
somethingobvious?
{beamer} {overlays }
editedJun6'13at9:16

askedJun6'13at9:10
user592748
28

3 Areyouawareofthe <+>syntaxwhichautomaticallyincrementstheoverlaynumber?percusseJun6'13at
9:42

1 Iamlookingitup.Buttheproblemisitnotsequential.Thenodesinthestatediagramneedtobehighlightedin
relationtoafunctionandhenceIneedtousenumberstoexplicitlystateit. user592748 Jun6'13at10:00

1 Youcanalsogiverelativeincrementstosyncdifferentelements.percusseJun6'13at10:01
1 Canyoupointtoanyexamples?Thanks! user592748 Jun6'13at10:10
1 Itwouldbeeasiertoanswerthisifyouaddedaminimalworkingexample(MWE)otherwiseit'shardtoknowwhat
willwork.LoopSpaceJun6'13at11:23

3 Answers
Here'sanexampleofusingincrementaloverlayspecificationswithoffsets.
\documentclass{beamer}
\begin{document}
\begin{frame}[label=integralofx]{Example:Integralof$f(x)=x$}
\begin{example}<+>
Find$\int_0^3x\,dx$.
\end{example}
\begin{solution}<+>
\action<.>{Forany$n$wehave$\alert<.(5)>{\Deltax=\frac{3}{n}}$andforeach
$i$between$0$and$n$,$\alert<.(4)>{x_i=\frac{3i}{n}}$.}
\action<+>{Foreach$i$,take$x_i$torepresent$f$onthe$i$thinterval.}
\action<+>{So}
\begin{align*}
\action<.>{\int_0^3x\,dx=\lim_{n\to\infty}R_n}
\action<+>{&=\lim_{n\to\infty}\sum_{i=1}^n\alert<.(1)>{f(x_i)}\,\alert<.
(2)>{\Deltax}}
\action<+>{=\lim_{n\to\infty}\sum_{i=1}^n
\alert<.>{\left(\frac{\alert<.(2)>{3}i}{\alert<.(2)>{n}}\right)}
\alert<+>{\left(\frac{\alert<.(1)>{3}}{\alert<.(1)>{n}}\right)}\\}
\action<+>{&=\lim_{n\to\infty}\alert<.>{\frac{9}{n^2}}\alert<.(1)>
{\sum_{i=1}^ni}}
\action<+>{=\alert<.(1)>{\lim_{n\to\infty}}\frac{9}{\alert<.(1)>{n^2}}
\cdot\alert<.>{\frac{\alert<.(1)>{n(n+1)}}{2}}}
\action<+>{=\frac{9}{2}\alert<.>{\cdot1}}
\end{align*}
\end{solution}
\end{frame}
\end{document}

http://tex.stackexchange.com/questions/117887/how-to-insert-overlays-in-latex-beamer

1/3

Das könnte Ihnen auch gefallen